The amount of energy it takes to lift a box might be a function of which of the
following?
A. The shape of the box
B. The weight of the objects inside the box
C. The time of day that you are lifting it

Answers

Answer 1

Answer:

A

Step-by-step explanation:

Because energy takes up the same inside the box?

Answer 2
It would be B because if there is more weight you would need to exert more energy to lift it

Related Questions

plz help i will give brainliest!!! 2.5(4x+10)−2(3x−15)

Answers

Answer:

4x+55

Step-by-step explanation:

Expand and simplify

10x+25x-6x+30

4x+55

Answer:

4x+55

Step-by-step explanation:

I used the PEDMAS to simplify the equation doing the parentheses first then addition and subtraction. The picture attached showed what order I did it in and I hope this really helped! Good LUCK on the rest of your homework.

What is -6 divided by -1.2?

Answers

Answer:

5.

Step-by-step explanation:

This is the same as saying:

-6/(-6/5).

To solve the equation, we multiply by the reciprocal and simplify. In this case, we multiply both sides of the equation by -5/6.

-6 * (-5/6) = 5.

The answer must be positive, as two negatives divided equal a positive.

I hope this helps!

The solution of expression after divide is,

⇒ 5

We have to given that;

⇒ - 6 is divided by - 1.2

Now, We can simplify as,

⇒ - 6 is divided by - 1.2

It can be written as,

⇒ - 6 ÷ - 1.2

⇒ - 6 / - 1.2

⇒ 6 / 1.2

⇒ 60 / 12

⇒ 5

Thus, The solution of expression after divide is,

⇒ 5

Learn more about the divide visit:

https://brainly.com/question/28119824

#SPJ6

please help as soon as possible
I WILL MARK YOU AS BRAINLIEST ​

Answers

Answer:

Last choice

Step-by-step explanation: Subtract 11 and then divide by 3. A would be greater than -2 and last choice shows that

Tiffany cells 2 kinds of homemade tomato sauce

Answers

Answer:

b

Step-by-step explanation:

Let x represent the number of quarts of Tuscan sauce and

y represents the number of quarts of marinara sauce Tiffany makes.

A quart of Tuscan sauce requires 6 tomatoes and 1 cup of oil

x quarts requires 6x tomatoes and 1x cups of oil

A quart of her marinara sauce requires 5 tomatoes and 1.25 cups of oil

y quarts requires 5y tomatoes and 1.25 y cups of oil

She has 45 tomatoes and 10 cups of oil on hand.

So the constraints are

6x+5y≤45

1x+1.25y≤10

x>=0 and y>=0

After drawing a diagram of what's required to make 1 quart of Tuscan and 1 quart of Marinara sauce, you can more clearly choose answer choice (B) 6x + 5y < (or equal to) 45, x + 5/4y < (or equal to) 10, x>(or equal to)0, y>(or equal to)0.

please mark brainliest :)

the answer is B !!! hope it’s correct

BEST ANSWER GETS TO CHOOSE BRAINLESST OR FOLLOW!​

Answers

Answer:13 is>

14 is =

15 is >

16 is<

17 is<

18 is =

Step-by-step explanation:

I just know plz mark brainliest

Line I and h intersect at what point

Answers

Answer:

please show where the lines are graphed in a picture

Step-by-step explanation:

14. Roger is on a playground swing, and he is swinging back and forth in such a way that the height, h, in feet, of the swing off the ground is given by the equation h=3cos(3π/2t) +5, where t is in seconds. How many seconds elapses between two consecutive times that the swing is at its maximum height?​

Answers

Answer:

The time [tex]t = \frac{3}{2}[/tex]  seconds elapses between two consecutive times that the swing is at its maximum height  'h' = 2

 Step-by-step explanation:

Explanation:-

Step(i):-

Given function [tex]h(t) = 3 cos (\frac{3\pi }{2 t} ) +5[/tex] ....(i)

By using derivative formulas

[tex]\frac{d cosx }{d x} = -sinx[/tex]

[tex]\frac{d x^{n} }{d x} = n x^{n-1}[/tex]

[tex]\frac{d t^{-1} }{d x} = -1 t^{-1-1} = - t^{-2} = \frac{-1}{t^{2} }[/tex]

Step(ii):-

Differentiating equation(i) with respective to 't'

[tex]h^{l} (t) = 3(-sin(\frac{3\pi }{2t})\frac{d}{dt} (\frac{3\pi }{2t } )+0[/tex]  ...(ii)

[tex]h^{l} (t) = 3(-sin(\frac{3\pi }{2t})(\frac{-3\pi }{2t^{2} } )+0[/tex]

Equating zero

[tex]h^{l} (t) = 3(-sin(\frac{3\pi }{2t})(\frac{-3\pi }{2t^{2} } )=0[/tex]

[tex]3(-sin(\frac{3\pi }{2t})(\frac{-3\pi }{2t^{2} } ) = 0[/tex]

on simplification , we get

[tex](sin(\frac{3\pi }{2t}) = 0[/tex]

now we use formulas

sin 0 = 0 and sinπ = 0

General solution

[tex](sin(\frac{3\pi }{2t}) = sin\pi[/tex]

[tex](\frac{3\pi }{2t}) = \pi[/tex]

Cancellation 'π' on both sides, we get

[tex]3 = 2 t[/tex]

Dividing '2' on both sides , we get

[tex]t = \frac{3}{2}[/tex]

Again differentiating with respective to 't' , we get  

[tex]h^{ll} (t) = 3(-cos(\frac{3\pi }{2t})(\frac{-3\pi }{2t^{2} } )+ (-3)(-sin(\frac{3\pi }{2t} )(\frac{6\pi }{2t^{3} }[/tex]

Put t= 3/2 and simplification

[tex]h^{ll} (t) < 0[/tex]

The maximum height

                   [tex]h(t) = 3 cos (\frac{3\pi }{2 t} ) +5[/tex]

               [tex]h(\frac{3}{2} ) = 3 cos (\frac{3\pi }{2(\frac{3}{2} )} )+5[/tex]

              [tex]h(\frac{3}{2} ) = 3 cos (\pi )+5 = -3+5 =2[/tex]

[tex]t = \frac{3}{2}[/tex]  seconds elapses between two consecutive times that the swing is at its maximum height  'h' = 2

Conclusion:-

The time [tex]t = \frac{3}{2}[/tex]  seconds elapses between two consecutive times that the swing is at its maximum height  'h' = 2

 

 


[tex]( {2}^{ - 1} + {3}^{ - 1} )^{2} [/tex]
solve.

will give the brainliest​

Answers

Answer:

25/36

Step-by-step explanation:

(2^-1 + 3^-1)^2

(1/2 + 1/3)^2

(5/6)^2 = 25/36

25/36 or .694

2^-1= 1/2
3^-1=1/3

Add the together
5/6 square it and you get 25/36

soooo i need help can yall find the answer for me I put mad points so better get it right .

Answers

Answer:

1.08 * 10 ^8

Step-by-step explanation:

2.4 * 10^4   * 4.5 * 10^3

Multiply the constants

10.8

Add the exponents

10^(4+3) = 10^7

Put together

10.8 * 10 ^7

Put we can only have one number in front of the decimal

Move the decimal one place to the left and add one to the exponent

1.08 * 10 ^8

What is the equation in slope-intercept form of a line with slope of 5 and y-intercept of 3?

Answers

y=5x+3

Because the starting value(y intercept is 3) and the slope(x) is 5

Becky made 4 quarts of chicken noodle soup in a big pot, then served 112-cup bowls of soup to 6 people. How much soup is left in the pot?

Answers

Answer:

1qt 3c

Step-by-step explanation:

A. 256
B. 265
C. 297
D. 279​

Answers

Answer:

A.

Step-by-step explanation:

So calculate one of the triangles.

[tex]\frac{12*8}{2} =48[/tex]

48 * 4 = 192

192 + 64 = 256

The area of the base of the cone is 64π square millimeters, and the area of the lateral surface is 112π square millimeters. Find the radius r and slant height ℓ of the cone.

Answers

Answer:

Radius = 8 mm

Slant Height = 14 mm

Step-by-step explanation:

Base of a cone is a circle.

Area of circle is given as [tex]A=\pi r^{2}[/tex]

Where [tex]r[/tex] is the radius.

Given that

[tex]A = 64 \pi\ mm^2\\\Rightarrow \pi r^{2} = 64\pi \\\Rightarrow r^{2} = 64\\\Rightarrow r = 8\ mm[/tex]

Hence, radius is 8 mm.

Formula for Lateral Surface Area of a cone:

[tex]LSA = \pi rl[/tex]

Where [tex]r[/tex] is the radius and

[tex]l[/tex] is the slant height of cone

Given that [tex]LSA = 112 \pi\ mm^2[/tex]

[tex]\Rightarrow 112\pi = \pi \times 8 \times l\\\Rightarrow 8 \times l = 112\\\Rightarrow l = 14\ mm[/tex]

Hence, slant height is 14 mm

evaluate the following expression -7x(7+9)

Answers

Answer:

-112

Step-by-step explanation:

Add

-7(7+9)

Multiply

-7x16

Wallah! You have the answer

-112

[tex]\text{Simplify the expression:}\\\\-7x(7+9)\\\\\text{Use the distributive property}\\\\-49x-63x\\\\\text{Combine like terms}\\\\\boxed{-112x}[/tex]

00:00
Brice is finding the sum of 468 and 241 by breaking it into smaller problems
He uses place value and finds the sums of the hundreds, tens, and ones.
What is the sum of the tens? Enter your answer in the box.
1​

Answers

468+241=709 so 0 is in the tenth place because 6+4= 10 and one gets carried over to the 4+2

what is the surface area of a rectangle 4/3 x 5/4 x 2/3

Answers

Answer:

surface area of rectangle= 2[(l+w) +(l+h). +(w+h)]

hope it helps

Soledad has a storage box. The box is 6 1/2 inches long, 4 3/4 inches wide and 7 inches tall. She wants to run a border around the top of the box. How much border does she need?

Answers

Answer:

She need 10.5 inches border

Step-by-step explanation:

Length of box =[tex]\frac{1}{2} inches[/tex]

Breadth of box =[tex]4\frac{3}{4} inches[/tex]

Breadth of box =[tex]\frac{19}{4} inches[/tex]

Height of box = 7 inches

Now we are given that Soledad wants to run a border around the top of the box.

So, Length of top = Perimeter = [tex]2(l+b)=2(\frac{1}{2}+\frac{19}{4})=10.5[/tex]

Hence She need 10.5 inches border

HELP ASAP it says my questions should be at least 20 characters long so what I’m saying right now doesn’t mean anything.

Answers

Answer: See pic above for answer. I got it from Photomath

Plz put brainliest



A new car is purchased for 23400 dollars. The value of the car depreciates at 11.5% per year. To the nearest year, how long will it be until the value of the car is 12700 dollars?

Answers

Answer: 5 years

Step-by-step explanation:

Using same formula as for Compound Interest:

12700 = 23400 (1-.115)^t

12700 = 23400 x 0.885^t

0.885^t = 127/234

Convert decimal into fraction:

177/200^t = 127/234

Take logarithm of both sides of thr equation:

t = log 177/200 (127/234)

t = 5.00242

Is the relation a function?
{(-6, -1), (5,-1), (0, -1), (-2, -1), (3, -1)}

Answers

Answer:

yes it is a function

Step-by-step explanation:

For that no two x values are the same with different y values

Arrange from smallest to largest
0.01, 1.01, 10.01, 1.001

Answers

Answer:

0.01 --> 1.001 ---> 1.01 ---> 10.01

The numbers in the ascending order:

0.01, 1.01, 1.001, 10.01.

What is ascending order?

Putting numbers in ascending order simply means to do it from smallest to largest.

Given:

0.01, 1.01, 10.01, 1.001.

The numbers from smallest to largest:

That means, in the ascending order.

So,

0.01, 1.01, 1.001, 10.01.

Therefore, 0.01, 1.01, 1.001, 10.01 are in the required order.

To learn more about the ascending order;

https://brainly.com/question/20681445

#SPJ2

What is the midpoint of the segment below? (2,3)(-3,-2)

Answers

Answer:

(-0.5, 0.5)

Step-by-step explanation:

If those two points are endpoints, just use the midpoint formula.

(-0.5, 0.5)

Answer:

(-1/2,1/2)

Step-by-step explanation:

To find the midpoint, add the x coordinates and divide by 2

(2+-3)/2 = -1/2

add the y coordinates and divide by 2

(3+-2)/2 = 1/2

The midpoint is (-1/2,1/2)

Which function is represented by the graph below?

Answers

Answer:

y= 2x-4

Step-by-step explanation:

The y-intercept is at -4, it is a positive slope, plotting from the y-axis rise over run is equal to 2/1 which means that the slope is 2x. giving you y= 2x-4

The equation which satisfy to the graph is Y= 2x- 4

What is an equation?

An equation is a mathematical expression that contains an equals symbol. Equations often contain algebra.

According to graph

There are multiples points given on which line was passing

But we will select only which is clear

1st point = (0, 2)

2nd point = (-4,0 )

Now we will put values on equations and check which will satisfy

For 1st equation

Y= 2x- 4

Checking with 1st point

Putting x= 2

Y= 0 which satisfy

Checking for 2nd point

Putting x=0

Y = -4 satisfy

Hence, equation which satisfy to graph is Y= 2x- 4

To know more about equation here :

https://brainly.com/question/24169758

# SPJ2

If f(x) = 2x + 1 and g(x) = x-2 what is the value of f(g(f(3)))? This is so confusing plz help me will mark brainliest A) 1 B) 3 C) 5 D) 7 E) 11

Answers

Answer:

11

Step-by-step explanation:

f(3) = 2*3+1 = 6+1 = 7

Then find g(7)

g(7) = 7-2 = 5

Then find f(5)

f(5) = 2*5 +1 = 10+1 = 11

f(g(f(3))) = 11

Answer:

11

Step-by-step explanation:

f(x)=2x + 1

f(3)= 2.3 + 1

f(3)=7

f(g(f(3))) = f(g(7))

Now, we have to find g(7)

g(x)=x -2

g(7)= 7 -2

g(7)=5

f(g(f(3))) = f(g(7)) = f(5)

now we have to find f(5)

f(x)=2x + 1

f(5)=2.5 + 1

f(5)=11

Hope this helps ^-^

PLEASE HELP WILL GIVE BRAINLIEST AND 20 POINTS

solve this system of equations

3y-5x=12
y=1/3x

Answers

Answer:

x=-3

y=-1

(-3,-1)

Answer:

(-3, -1).

Step-by-step explanation:

I have attached the work to your problem.

Please see the attachment below.

I hope this helps!

Find the solution(s) to x^2- 14x + 49 = 0.
O A. x=-2 and x = 7
B. x= -1 and x = 14
C. x= 7 only
D. x = 7 and x = -7

Answers

Answer:

[tex] \boxed{C. \: x = 7 \: only} [/tex]

Step-by-step explanation:

[tex] = > {x}^{2} - 14x + 49 = 0 \\ \\ = > {x}^{2} - (7 + 7)x + 49 = 0 \\ \\ = > {x}^{2} - 7x - 7x + 49 = 0 \\ \\ = > x(x - 7) - 7(x - 7) = 0 \\ \\ = > (x - 7)(x - 7) = 0 \\ \\ = > {(x - 7)}^{2} = 0 \\ \\ = > x - 7 = 0 \\ \\ = > x = 7[/tex]

I need help with is question ASAP please

Answers

The y-intercept looks like it's 0 and the slop looks like 6/2 or 3 or maybe 7/2

Which equation represents a line parallel to the line whose equations is -2x + 3y =
-4 and passes through the point (1,3)?

Answers

Answer:

2.  y - 3 = 2/3 (x - 1).

Step-by-step explanation:

-2x + 3y = -4

3y = 2x - 4

y = 2/3 x - 4/3 - so the slope is  2/3.

The slope of a line parallel to it is also 2/3.

It also passes through the point (1, 3).

Using the point-slope form of  a line:

y - y1 = m(x - x1) where m = the slope and (x1, y1) is a point on the line, we have:

y - 3 = 2/3 (x - 1)  <--- is the required equation.

Answer:

2

Step-by-step explanation:

parallel lines have same slope with different intercept

y= mx+b

m is going to be same with different b

so the given function is:

-2x+3y=-43y= 2x-4y= 2/3x - 4/3

Given options:

1. y-3= - 2/3(x-1)  ⇒ y= -2/3x +3 + 2/3 ⇒ y= -2/3x +11/3

it has different slope, so is not parallel

2. y-3= 2/3(x-1) ⇒ y= 2/3x+3-2/3 ⇒ y= 2/3x + 7/3

it has same slope, so is parallelit should be passing through point (1,3)3= 2/3+7/3 ⇒ 3=3, yes it does

3. y-3= -2/3(x+1) ⇒ y= - 2/3x +3- 2/3 ⇒ y= -2/3x + 7/3

it has different slope, is not parallel

4. y-3= 2/3(x+1) ⇒ y= 2/3x +3+ 2/3 ⇒ y= 2/3x +11/3

it has same slope, so is parallelit should be passing through point (1,3)3= 2/3+11/3 ⇒ 3≠13/3, no it doesn't

Solve:
-3(7p + 5) = 27
Helpppp

Answers

Answer:

-2

Step-by-step explanation:

-21p - 15 = 27

-21p = 42

p = -2

Answer:

-2

Step-by-step explanation:

-21p - 15 = 27

-21p = 42

p = -2

Helppppp meeeee answer thissssssss

Answers

Answer:

(-1.025)^3 x (-1.025)^2=(- 1.025)^5

(-y)^3 x (-y)^2 x (-y)= y^6

5= odd, - stays

6= even, - becomes +

Answer:

5) (-1.025)^3* (-1.025)^2 = (-1.025)^5 = (-41/40)^5

10) -y^3 * -y^2 * -y

Multiplying an odd number of negative terms makes the product negative

= - y^3 * y^2 * y

= - y^6

Other Questions
Is the earth flat? Or is the government just saying it isnt flat to lie to us ? What is weather?A. Whats happening in space at a time and placeB. Whats happening in our atmosphere at a time and placeC. The average weather over many yearsD. The average temperature over many years a(n) ______ reverses the flow of current through an electric motor.brushbatteryarmaturecommutator pls help ASAP for brainliest answer Suppose the high tide in Seattle occurs at 1:00 a.m. and 1:00 p.m. at which time the water is 10 feet above the height of the low tide. Low tides occur 6 hours after high tides. Suppose there are two high tides and two low tides every day and the height of the tide varies sinusoidally. a) Find a formula for the function y = h(t) that computes the height of the tide above low tide at time t. (In other words, y = 0 corresponds to low tide) b) What is the tide height at 11:00 am? during your lifetime, you will produce enough saliva to fill two swimming pools What is the decimal equivalent of 23/9 CAN SOMEONE PLZ HELP ME GET ONE MORE BRAINLIEST ANSWER??? The ratio of boys to girls in a club is 4:3. If there are 48 boys, how many members of the club are there? using the letter A what is the genotype of an organism that is homozygous dominate?a. AAb. Aac. aad AaA What does the film Inside Out reveal about identity? * ANSWER ASAP * What is the answer?? It's pretty difficult. Based on their electronegativities, you would expect ____ elements to pull on electrons more strongly than ______ elements. What is the purpose of a fitness log?to apoly the HIT principleto limt the amount of time spent on varying activitiesC to track and monitor progressD to increase personal interest in activitiesPlease select the best answer from the choices provided Find the point estimate for the true difference between the given population means.Weights (in Grams) of Soap Bar A: 121, 122, 124, 123, 120, 124, 121, 121, 121, 123, 120 Weights (in Grams) of Soap Bar B: 121, 120, 122, 119, 121, 122, 122, 120, 120, 121, 122, 123, 119 hiii guys i need help with my homewrok [tex]-9\leq 7-8x[/tex] I NEED HELP, AND FAST An employee receives an hourly rate of $15, with time and a half for all hours worked in excess of 40 during the week. Payroll data for the first week of the calendar year are as follows: hours worked, 46; federal income tax withheld, $110; Social security tax rate, 6%; and Medicare tax rate, 1.5%; state unemployment tax, 5.4% on the first $7,000; federal unemployment tax, 0.8% on the first $7,000. What is the net amount to be paid to the employee? If required, round your answers to the nearest cent. Health department officials are trying to identify the source of an outbreak of an intestinal virus. Which is the most likely source of thevirus?O A. polluted airO B. contaminated waterOC. poor nutritional habitsO D. lack of exercise You have 1200 feet of fencing and you want to make two fenced in enclosures by splitting one enclosure inhalf. What are the largest dimensions of this enclosure that you could build?